It gets a bit weird

a 1 2 + a 2 2 + + a 19 2 = a 20 2 + + a 37 2 a_1^2+a_2^2+\cdots +a_{19}^2=a_{20}^2+\cdots +a_{37}^2

Do there exist consecutive integers a 1 , , a 37 a_1,\ldots ,a_{37} such the equation above holds? If so, enter the largest possible value of a 1 a_1 . If you come to the conclusion that no such integers exist, enter 999.


The answer is 666.

This section requires Javascript.
You are seeing this because something didn't load right. We suggest you, (a) try refreshing the page, (b) enabling javascript if it is disabled on your browser and, finally, (c) loading the non-javascript version of this page . We're sorry about the hassle.

2 solutions

Rishabh Jain
Jan 3, 2016

a 2 = a 1 + 1 a 3 = a 1 + 2 a_2=a_1+\color{#D61F06}{1}\\ a_3=a_1+\color{#D61F06}{2}\\ and so on (as they are consecutive integers) a 1 2 + a 2 2 + . . . + a 19 2 = a 20 2 + . . . + a 37 2 a_1^2+a_2^2+...+a_{19}^2=a_{20}^2+...+a_{37}^2 a 1 2 + ( a 1 + 1 ) 2 . . . . . + ( a 18 + 1 ) 2 = ( a 19 + 1 ) 2 + ( a 20 + 1 ) 2 . . . . ( a 36 + 1 ) 2 \Rightarrow a_1^2+(a_1+1)^2.....+(a_{18}+1)^2=(a_{19}+1)^2+(a_{20}+1)^2....(a_{36}+1)^2 a 1 2 + 2 a 1 ( 1 + 2.. + 18 19 20.... 36 ) + ( 1 2 + 2 2 + . . . . 1 8 2 1 9 2 2 0 2 . . . . 3 6 2 ) = 0 \Rightarrow a_1^2+2a_1(1+2..+18-19-20....-36)+(1^2+2^2+....18^2-19^2-20^2-....-36^2)=0 a 1 2 + 2 a 1 ( 2 ( i = 1 18 i ) ( i = 1 36 i ) ) + ( 2 ( i = 1 18 i 2 ) ( i = 1 36 i 2 ) ) \Rightarrow a_1^2+2a_1(\color{#20A900}{2(\displaystyle \sum_{i=1}^{18} i)-(\displaystyle \sum_{i=1}^{36}i)})+(\color{#3D99F6}{2(\displaystyle \sum_{i=1}^{18}i^2)-(\displaystyle \sum_{i=1}^{36}i^2)})\\ = a 1 2 648 a 11988 = 0 =a_1^2-648a-11988=0 a 1 = 18 , 666 a_1=-18,666

Largest possible value of a 1 a_1 = 666 \color{magenta}{666}\\

Yes, very nicely done! (+1)

Otto Bretscher - 5 years, 5 months ago

Got the same equation. But did it with a different method.

Shubhang Mundra - 5 years, 5 months ago

About the same method. Got the same equation at last. Some equation though I gotta say

Shreyash Rai - 5 years, 5 months ago

I took advantage of the near-symmetry of the equation and set a 19 = : n a_{19} =: n , so that n 2 + k = 1 18 ( n k ) 2 = k = 1 18 ( n + k ) 2 . n^2 + \sum_{k = 1}^{18} (n-k)^2 = \sum_{k=1}^{18} (n+k)^2. In ( n ± k ) 2 = n 2 + k 2 ± 2 n k (n\pm k)^2 = n^2 + k^2 \pm 2 nk , the squares cancel each other on both sides so that we simply get n 2 = k = 1 18 4 n k . n^2 = \sum_{k=1}^{18} 4nk. One obvious solution is n = 0 n = 0 , which would give a 1 = 18 a_1 = -18 . The other solution is positive: n = 4 k = 1 18 k = 4 19 18 2 = 684. n = 4\sum_{k=1}^{18} k = 4\cdot \frac{19\cdot 18}2 = 684. Finally, a 1 = a 19 18 = n 18 = 666 a_1 = a_{19} - 18 = n - 18 = \boxed{666} .

I took advantage of the fact that @Otto Bretscher tends to reserve 666 (rather than 999) for the nonexistence of an answer. :P

Andrew Ellinor - 5 years, 5 months ago

Yes, exactly! (+1) That's how I tackled the more general problem . It's always good to go for a "symmetry bonus"

Otto Bretscher - 5 years, 5 months ago

Log in to reply

Ah, yes. Thanks for pointing me to that problem. I solved that the same way you did, even choosing the same letter m m for n + k n + k .

Arjen Vreugdenhil - 5 years, 5 months ago

0 pending reports

×

Problem Loading...

Note Loading...

Set Loading...